Convergence intégrale

Bonjour,

Je "traite" un petit exercice d'intégrale indéfinie:

En 1) je dois déterminer la nature de: $$I =\int_{0}^{+\infty} \dfrac{arctan(t)}{t} \mathrm{d}t $$
Ce que j'ai fait comme ceci :
D'abord qq soit t >0 on a que f(t) = Arctan(t)/t = 0 et on peut réaliser une prolongation par continuité en t = 0 avec f(t) = 1.

La lim en +inf de Arctan(t) = Pi/4, et qd t tend vers + Inf alors I ˜ K/t avec k = Pi/2. C'est un équivalent de signe constant positif. L'intégrale de 1/t (entre 1 et + inf) diverge. Le critère de comparaison par équivalence s'applique et on a :
L'intégrale de f(t)dt (entre 1 et + inf) diverge et l'intégrale de f(t)dt (entre 0 et + inf) diverge. Rem: entre 1 et + inf I se calcule bien sur (on a F = Pi/2.ln(x) .. ) mais comme je dois prendre en compte la borne inf = 0 je ne peux pas l'utiliser).

En 2), je dois déterminer la nature de:
$$J = \int_{0}^{+\infty} \dfrac {arctan(t)-\dfrac{\pi}{2} } {t} \mathrm{d}t $$
Heu .. là j'ai un doute: puis-je "simplement" dire que en +inf l'intégrale J = Pi/2t - Pi/2t .. soit 0 ?

En 3), je dois déterminer la nature de:
$$K = \int_{0}^{+\infty} \dfrac{arctan(t)} {t} - \dfrac{\pi}{2(1+t)} \mathrm{d}t $$
a) Montrer que la fonction à intégrer g(t) est prolongeable par continuité en 0: je trouve que g est prolongeable en posant g(0) = 1- pi/2

b) En utilisant la formule valable pour tous x>0 : Arctan(t) + Arctan(1/t) = Pi/2 effectuer un développement limité d'ordre 2 de f au voisinage de l'infini et en déduite que K est convergente.

Un avis/commentaire serait bienvenu sur la méthode : On connait un DL de Arctan au voisinage de 0. Si x tend vers + inf alors 1/x tend vers 0 et on va utiliser le DL en 0 de "Pi/2 - Arctan(1/t)" puisque Pi/2 - Arctan(1/t)= Arctan(t). Il faut alors calculer le DL à l'ordre 2 de g(t) et regarder la lim en + inf du DL obtenu pour voir si on a bien une limite finie .. ?.

E

Réponses

  • Tu as eu une réponse ici, pourquoi reposer exactement le même message initial ????
  • Pour la question 2):

    - Tout d'abord, j'ai bien vérifié, et les bornes de l'intégrale J sont bien 0 et + Inf.

    En + Inf: pour le numérateur on a lim (Arctan(x) - Pi/2) = 0 et donc lim de J = 0/(+ inf) = 0.

    En 0+: Je divise l'expression dans l'intégrale J par t : $$ J = \dfrac {\dfrac{arctan(t)}{t}-\dfrac{\dfrac{\pi}{2}}{t}} {\dfrac{t}{t}} $$
    J'ai alors: $$ J = \dfrac {arctan(t)} {t} -\dfrac{\pi}{2t}$$
    Lim de (J) en 0+ = Lim en 0+ de Arctan(t)/t - Lim en 0+ de (Pi/2t) avec : Lim en 0+ de Arctan(t)/t = 1 et lim en 0+ de (Pi/2t) = + Inf.

    Donc Lim de (J) en 0+ = 1 - Inf = - Inf. Et donc l'intégrale est .... divergente. Et même en 0+ on aurait alors Lim = +Inf. C'est un peu bizarre.
  • C'est pour cela que je posais la question.
    Alors l'idée de l'énoncé est probablement que, après avoir vu la divergence due à la limite non nulle, on essaie de supprimer cela en soustrayant brutalement la limite, donc on a le cas de la question 2. Comme ça ne marche pas, on passe à une soustraction plus élaborée, de façon qu'il n'y ait plus de problème en 0 (t+1) mais que la fonction à intégrer ne soit plus équivalente à $\frac a t$.

    Cordialement.
  • Oui cela doit être ça ... Pour mieux comprendre j'ai tracé les courbes de chaque fonction des intégrales J et K.

    Sur le graphe (en pj) la fonction de l'intégrale J est en bleu et la fonction de l'intégrale K est en rouge. On voit bien que la 2ième fonction "réduit la zone de divergence aux alentours de 0" de la 1ière fonction et que la courbe tend vers 0.

    Par contre j'ai un pb : Comme demandé j'utilise la relation "Arctan(t) + Arctan(1/t) = Pi/2" pour calculer le DL en + oo de la fonction $$f(t) = \dfrac{arctan(t)} {t} - \dfrac{\pi}{2(1+t)} $$. Cela bien sûr pour calculer la lim de f(f) en +oo.
    Je tombe sur un résultat différent du calcul du DL sans utilisation de la relation ci-dessus ! C'est bêta mais il y a un gros truc qui m'échappe.

    ==> Avec la relation sur Arctan j'ai ceci:
    Arctan(t) = Pi/2 - Arctan(1/t) et qd t tend vers +oo 1/t tend vers 0 et Arctan (1/t) tend vers 0. Donc en + oo on a Arctan(t) = Pi/2 (résultat connu !). Et : Arctan(t)/t = Pi/2t.

    Je calcule le DL, à l'ordre 2, de $$\dfrac{\pi}{2(1+t)}$$ en utilisant le DL en 0 de 1/(1 + X) avec ici X = 1/t puisque on est en + oo. J'ai alors ceci : Pi/2t - (Pi/2 - Pi/2t + Pi/2t^2 + ....o(1/t^2) ) = -Pi/2 + Pi/2t^2 + ....o(1/t^2). Et donc la lim en +oo = -Pi/2 ce qui est faux d'après la courbe de f(t).

    ==> Sans la relation, via un calcul "direct" j'ai ceci: On pose X = 1/t (chg de variable pour +oo) et
    DL, ordre 2 de Arctan(X)/X :
    DL de Arctan(X) en 0 = X - (X^3)/3 + ... + o(X^3)
    et DL de Arctan(X)/X = 1 - (X^2)/3 + ... + o(X^2) = a
    Pour le 2iem terme de f(t):
    DL de (1/1+X) = (1 - X + X^2 + .. + o(X^2))
    et DL de Pi/2.(1/1+X) = (Pi/2 - Pi/2.X + Pi/2. X^2 + .. + o(X^2)) = b

    Lorsque je fais la différence a - b des DL (ce sont des DL, donc "j'ai le droit"):
    (1 - (X^2)/3 + ... + o(X^2)) - (Pi/2 - Pi/2.X + Pi/2. X^2 + .. + o(X^2)) = 1 - Pi/2 + Pi/2.X - (1/3+Pi/2)X^2 + ..
    En remplaçant X = 1/t :
    a - b = 1 - Pi/2 + Pi/2t - (1/3+Pi/2). 1/t^2 ..
    Et donc la lim en +oo = 1 - Pi/2 ce qui est faux d'après la courbe de f(t) mais surtout est différent du calcul .
    précédent ! Je pifomètre une erreur élémentaire de ma part (sur le terme en Arctan) ...

    Bon, en tous cas f(t) a une limite finie en + oo et donc l'intégrale K est convergente ...

    E69762
  • Heu ... ton DL de $\dfrac{\pi}{2(1+t)}$ me paraît bien compliqué. Quand $t\to +\infty, \, \dfrac{\pi}{2(1+t)}\sim \dfrac{\pi}{2t}$, qui est donc le terme d'ordre 1 de ton développement asymptotique. Et le terme d'ordre 0 est 0 (la limite à l'infini de $\dfrac{\pi}{2(1+t)}$). Tu as dû mélanger $t$ avec $\frac 1 t$.

    Cordialement.
  • Bonsoir,

    je relance ce sujet car j'ai un dl de Pi/2t - 1/t², ça vous parle?
  • Quelqu'un pour m'aider ?
  • Bonjour,

    Je veux bien aider, mais peux-tu écrire la question précise ?
Connectez-vous ou Inscrivez-vous pour répondre.